The ASE Certification Test, specifically the A1 to A8 tests, is a comprehensive examination designed to validate the technical knowledge, skills, and abilities of automobile technicians. Administered by the National Institute for Automotive Service Excellence (ASE), these tests are a critical step towards achieving Master Automobile Technician certification, a prestigious title that significantly enhances your professional credibility in the automotive industry.
The A1 to A8 tests cover eight different areas of automotive repair and service, each focusing on a specific aspect of a vehicle’s system. These areas include:
Test Series | Description |
---|---|
A1 – Engine Repair | Covers engine repair skills |
A2 – Automatic Transmission/Transaxle | Covers automatic transmission and transaxle skills |
A3 – Manual Drive Train and Axles | Covers manual drive train and axles skills |
A4 – Suspension and Steering | Covers suspension and steering skills |
A5 – Brakes | Covers brake system skills |
A6 – Electrical/Electronic Systems | Covers electrical and electronic systems skills |
A7 – Heating and Air Conditioning | Covers heating and air conditioning skills |
A8 – Engine Performance | Covers engine performance skills |
Each test consists of 40 to 50 multiple-choice questions that assess your understanding of the principles, diagnostic methods, and repair techniques relevant to each area. The questions are designed to reflect real-world scenarios that automotive technicians encounter in their day-to-day work, making the ASE Certification Test a practical and valid assessment tool.
Passing the A1 to A8 tests not only demonstrates your technical proficiency but also signals to employers, colleagues, and customers that you are committed to maintaining a high standard of professionalism and quality in your work. The ASE Certification is widely recognized and respected within the automotive industry, making it a valuable asset for any automotive technician seeking to advance their career.
Did you know?
The ASE Certification Test, specifically the A1 to A8 tests, is a computer-based exam requiring deep automotive knowledge. Successful candidates typically have two years of hands-on experience or a combination of work experience and formal education. The tests are scored on a 0-100 scale, with 70 as the passing score. Employers often use these scores as a benchmark for hiring and promoting technicians, recognizing the certification as a mark of professional commitment and expertise.
A1-A8 Question Types Explained
The ASE Certification Test, specifically the A1 to A8 tests, assesses a broad range of skills and knowledge across eight key areas of automotive repair and service. Here’s a brief overview of each test:
- A1 – Engine Repair: This test measures your knowledge of general engine diagnosis, cylinder head and valve train diagnosis and repair, engine block diagnosis and repair, and lubrication and cooling systems diagnosis and repair. Proficiency in these areas requires a solid understanding of engine components and their functions, as well as the ability to diagnose and fix engine-related problems.
- A2 – Automatic Transmission/Transaxle: This test assesses your understanding of general transmission/transaxle diagnosis, in-vehicle transmission/transaxle maintenance and repair, off-vehicle transmission/transaxle repair, and more. To excel in this area, you need to be familiar with the components and operation of automatic transmissions/transaxles and be able to diagnose and repair related issues.
- A3 – Manual Drive Train and Axles: This test evaluates your knowledge of clutch diagnosis and repair, transmission diagnosis and repair, transaxle diagnosis and repair, drive shaft/half-shaft diagnosis and repair, and more. Proficiency in these areas requires a thorough understanding of the components and operation of manual drive trains and axles, as well as the ability to diagnose and repair related problems.
- A4 – Suspension and Steering: This test measures your understanding of steering systems diagnosis and repair, suspension systems diagnosis and repair, wheel alignment diagnosis, adjustment, and repair, and wheel and tire diagnosis and service. To perform well on this test, you need to be familiar with the components and operation of steering and suspension systems and be able to diagnose and repair related issues.
- A5 – Brakes: This test assesses your knowledge of hydraulic, drum, disc, power assist units, and anti-lock brake systems. Proficiency in these areas requires a solid understanding of brake system components and their functions, as well as the ability to diagnose and fix brake-related problems.
- A6 – Electrical/Electronic Systems: This test evaluates your understanding of general electrical system diagnosis, battery and starting system diagnosis and repair, charging system diagnosis and repair, and more. To excel in this area, you need to be familiar with the components and operation of vehicle electrical systems and be able to diagnose and repair related issues.
- A7 – Heating and Air Conditioning: This test measures your knowledge of A/C and heating systems diagnosis and repair, refrigeration system component diagnosis and repair, and operating systems and related controls diagnosis and repair. Proficiency in these areas requires a thorough understanding of heating and air conditioning systems and the ability to diagnose and repair related problems.
- A8 – Engine Performance: This test assesses your understanding of general engine performance diagnosis, ignition system diagnosis and repair, fuel, air induction, and exhaust systems diagnosis and repair, and more. To perform well on this test, you need to be familiar with the components and operation of engine performance systems and be able to diagnose and repair related issues.
Each of these tests requires a combination of theoretical knowledge and practical skills, as well as a problem-solving mindset to diagnose and repair automotive issues effectively.
Test | Name | Number of Certification Test Questions | Certification Test Time | Number of Recertification Test Questions | Recertification Test Time |
---|---|---|---|---|---|
A1 | Engine Repair | 60* | 1 1/4 hrs | 25 | 30 mins |
A2 | Automatic Transmission / Transaxle | 60* | 1 1/4 hrs | 25 | 30 mins |
A3 | Manual Drive Train and Axles | 50* | 1 hr | 20 | 30 mins |
A4 | Suspension and Steering | 50* | 1 hr | 20 | 30 mins |
A5 | Brakes | 55* | 1 1/4 hrs | 23 | 30 mins |
A6 | Electrical / Electronic Systems | 60* | 1 1/2 hrs | 25 | 45 mins |
A7 | Heating and Air Conditioning | 60* | 1 1/4 hrs | 25 | 30 mins |
A8 | Engine Performance | 60* | 1 1/4 hrs | 25 | 45 mins |
*Note: Certification tests include an extra 10 research questions that do not contribute to the final score. Since it’s not clear which questions are research questions, it’s important to answer all questions. To be eligible for the shorter recertification test, you must have previously passed the certification test.
The ASE certs are basically specialty knowledge in some branch of repair, such as automatic transmissions, electrical diagnostics, etc. They’re also grouped into major categories and some categories have advanced tests that require passing basic tests to even be allowed to take. For example, passing the eight A-series tests for passenger cars and light trucks qualifies you as a “Master,” and opens the door for advanced certifications (the three L-series tests) with tighter specializations.
Source: WebMaka/reddit
A1 – Engine Repair Test
The A1 Engine Repair Test is one of the eight tests in the ASE Certification series for Master Automobile Technician certification. It is designed to assess a technician’s knowledge and skills related to engine repair in automobiles.
The test consists of 60 multiple-choice questions, with an additional 10 research questions that do not count towards the final score. The time allotted for the test is 1 1/4 hours.
The A1 Engine Repair Test covers several key areas:
- General Engine Diagnosis: This section tests your ability to diagnose general engine problems. This includes identifying abnormal engine noises, checking engine condition using a vacuum gauge, compression gauge, and pressure gauge, and diagnosing engine mechanical, fuel, and ignition system related driveability problems.
- Cylinder Head and Valve Train Diagnosis and Repair: This section assesses your knowledge of cylinder head and valve train components. You’ll need to diagnose problems related to these components and perform necessary repairs, including valve train noise, oil leaks, and cylinder head warpage.
- Engine Block Diagnosis and Repair: In this section, you’ll be tested on your ability to diagnose and repair engine block problems. This includes diagnosing oil leaks, coolant leaks, and problems with the engine’s rotating and reciprocating assemblies.
- Lubrication and Cooling Systems Diagnosis and Repair: This section evaluates your understanding of the engine’s lubrication and cooling systems. You’ll need to diagnose problems such as overheating, coolant leaks, oil pressure problems, and abnormal oil consumption.
To perform well on the A1 Engine Repair Test, you need a solid understanding of engine components and their functions, as well as practical skills in diagnosing and repairing engine-related problems. It’s also important to have a thorough understanding of safety procedures and the use of service manuals and other technical literature.
Table: Engine Repair (Test A1) Content Areas
Content Area | Questions in Test | Percentage of Test |
---|---|---|
A. General Engine Diagnosis | 15 | 30% |
B. Cylinder Head and Valve Train Diagnosis and Repair | 10 | 20% |
C. Engine Block Diagnosis and Repair | 10 | 20% |
D. Lubrication and Cooling Systems Diagnosis and Repair | 8 | 16% |
E. Fuel, Electrical, Ignition, and Exhaust Systems Inspection and Service | 7 | 14% |
Total | 50 | 100% |
A1 Sample Question
A customer complains of a high-pitched squealing noise coming from the engine compartment that increases in frequency as the engine speed increases. The MOST likely cause of this noise is:
- A failing water pump.
- A loose or worn drive belt.
- A malfunctioning power steering pump.
- A clogged fuel injector.
Answer: B. A loose or worn drive belt.
Explanation:
A high-pitched squealing noise that increases with engine speed is most commonly associated with a loose or worn drive belt (Answer B). When the belt slips on the pulleys, it can produce a squealing noise.
Answer A is incorrect because a failing water pump typically causes a grinding or growling noise, not a high-pitched squeal.
Answer C is incorrect because a malfunctioning power steering pump usually causes a whining noise, especially when turning the steering wheel, not a squeal that increases with engine speed.
Answer D is incorrect because a clogged fuel injector would affect engine performance and might cause a misfire or rough idle, but it wouldn’t cause a squealing noise.
A2 – Automatic Transmission/Transaxle Test
The A2 Automatic Transmission/Transaxle Test is a part of the ASE Certification series for Master Automobile Technician certification. This test is designed to evaluate a technician’s knowledge and skills in the area of automatic transmission and transaxle systems in automobiles.
The test consists of 60 multiple-choice questions, with an additional 10 research questions that do not count towards the final score. The time allotted for the test is 1 1/4 hours.
The A2 Automatic Transmission/Transaxle Test covers several key areas:
- General Transmission/Transaxle Diagnosis: This section assesses your ability to diagnose general transmission and transaxle problems. This includes identifying abnormal noises and vibrations, checking fluid condition, diagnosing electronic control systems, and interpreting scan tool data.
- In-Vehicle Transmission/Transaxle Maintenance and Repair: This section tests your skills in performing in-vehicle repairs and maintenance. This includes verifying complaints, performing pressure tests, and diagnosing fluid leaks.
- Off-Vehicle Transmission/Transaxle Repair: In this section, you’ll be tested on your ability to perform off-vehicle transmission and transaxle repairs. This includes disassembling, inspecting, and reassembling transmission and transaxle components, as well as diagnosing and repairing hydraulic, mechanical, and electrical systems.
To perform well on the A2 Automatic Transmission/Transaxle Test, you need a solid understanding of automatic transmission and transaxle components and their functions, as well as practical skills in diagnosing and repairing transmission and transaxle-related problems. It’s also important to have a thorough understanding of safety procedures and the use of service manuals and other technical literature.
Table: Automatic Transmission/Transaxle (Test A2) Content Areas
Content Area | Number of Questions | Percentage of Test |
---|---|---|
A. General Transmission/Transaxle Diagnosis | 25 | 50% |
– Mechanical/Hydraulic Systems | (11) | – |
– Electronic Systems | (14) | – |
B. In-Vehicle Transmission/Transaxle Maintenance and Repair | 12 | 24% |
C. Off-Vehicle Transmission/Transaxle Repair | 13 | 26% |
– Removal and Installation | (4) | – |
– Disassembly and Assembly | (5) | – |
– Friction and Reaction Units | (4) | – |
Total | 50 | 100% |
A2 Sample Question
A vehicle with an automatic transmission is not shifting into third gear. The fluid level is correct and the fluid is not burnt. The MOST likely cause of this problem is:
- A worn clutch disc.
- A faulty shift solenoid.
- A clogged fuel filter.
- A malfunctioning engine temperature sensor.
Answer: B. A faulty shift solenoid.
Explanation:
Automatic transmissions use shift solenoids to control the flow of hydraulic fluid for shifting. If a vehicle is not shifting into a particular gear, a faulty shift solenoid (Answer B) is a likely cause.
Answer A is incorrect because automatic transmissions do not use a clutch disc. This is a component found in manual transmissions.
Answer C is incorrect because a clogged fuel filter would typically cause engine performance problems, not specific transmission shifting issues.
Answer D is incorrect because while a malfunctioning engine temperature sensor can cause various issues, it is not typically associated with a transmission failing to shift into a specific gear.
A3 – Manual Drive Train and Axles Test
The A3 Manual Drive Train and Axles Test is part of the ASE Certification series for Master Automobile Technician certification. This test is designed to evaluate a technician’s knowledge and skills in the area of manual drive train and axle systems in automobiles.
The test consists of 50 multiple-choice questions, with an additional 10 research questions that do not count towards the final score. The time allotted for the test is 1 hour.
The A3 Manual Drive Train and Axles Test covers several key areas:
- Clutch Diagnosis and Repair: This section assesses your ability to diagnose and repair clutch-related problems. This includes identifying abnormal noises and vibrations, checking hydraulic systems, and diagnosing clutch pedal linkage issues.
- Transmission Diagnosis and Repair: This section tests your skills in diagnosing and repairing manual transmission problems. This includes diagnosing gear noise and shift irregularities, inspecting and repairing transmission components, and understanding transmission gear ratios.
- Transaxle Diagnosis and Repair: In this section, you’ll be tested on your ability to diagnose and repair manual transaxle problems. This includes diagnosing gear noise and shift irregularities, inspecting and repairing transaxle components, and understanding transaxle gear ratios.
- Drive Shaft/Half-Shaft Diagnosis and Repair: This section evaluates your understanding of drive shaft and half-shaft components and their functions. You’ll need to diagnose problems such as abnormal noises and vibrations, and perform necessary repairs.
- Drive Axle Diagnosis and Repair: This section tests your ability to diagnose and repair drive axle problems. This includes diagnosing noise and vibration issues, inspecting and repairing axle shafts, and understanding axle gear ratios.
To perform well on the A3 Manual Drive Train and Axles Test, you need a solid understanding of manual drive train and axle components and their functions, as well as practical skills in diagnosing and repairing drive train and axle-related problems. It’s also important to have a thorough understanding of safety procedures and the use of service manuals and other technical literature.
Table: Manual Drive Train and Axles (Test A3) Content Areas
Content Area | Number of Questions | Percentage of Test |
---|---|---|
A. Clutch Diagnosis and Repair | 6 | 16% |
B. Transmission Diagnosis and Repair | 7 | 17% |
C. Transaxle Diagnosis and Repair | 7 | 17% |
D. Drive Shaft/Half-Shaft and Universal Joint/Constant Velocity (CV) Joint Diagnosis and Repair (Front and Rear Wheel Drive) | 5 | 13% |
E. Drive Axle Diagnosis and Repair | 7 | 17% |
F. Four-Wheel Drive/All-Wheel Drive Component Diagnosis and Repair | 8 | 20% |
Total | 40 | 100% |
A3 Sample Question
A vehicle with a manual transmission pops out of first gear shortly after being engaged. The transmission fluid level is correct and the fluid is not burnt. The MOST likely cause of this problem is:
- A worn first gear synchronizer.
- A faulty clutch master cylinder.
- A clogged fuel filter.
- A malfunctioning engine temperature sensor.
Answer: A. A worn first gear synchronizer.
Explanation:
Manual transmissions use synchronizers to smoothly engage the gears. If a vehicle pops out of first gear shortly after being engaged, a worn first gear synchronizer (Answer A) is a likely cause.
Answer B is incorrect because a faulty clutch master cylinder would typically cause difficulty in shifting gears, not the transmission popping out of a specific gear.
Answer C is incorrect because a clogged fuel filter would typically cause engine performance problems, not specific transmission shifting issues.
Answer D is incorrect because while a malfunctioning engine temperature sensor can cause various issues, it is not typically associated with a transmission popping out of a specific gear.
A4 – Suspension and Steering Test
The A4 Suspension and Steering Test is part of the ASE Certification series for Master Automobile Technician certification. This test is designed to evaluate a technician’s knowledge and skills in the area of suspension and steering systems in automobiles.
The test consists of 50 multiple-choice questions, with an additional 10 research questions that do not count towards the final score. The time allotted for the test is 1 hour.
The A4 Suspension and Steering Test covers several key areas:
- Steering Systems Diagnosis and Repair: This section assesses your ability to diagnose and repair problems related to steering systems. This includes identifying abnormal noises and vibrations, checking power steering fluid and pressure, diagnosing steering gear issues, and repairing or replacing steering linkage components.
- Suspension Systems Diagnosis and Repair: This section tests your skills in diagnosing and repairing suspension system problems. This includes diagnosing issues related to vehicle ride height, abnormal tire wear, vehicle handling, and noises or vibrations. It also involves repairing or replacing suspension system components such as control arms, springs, dampers, and bushings.
- Wheel Alignment Diagnosis, Adjustment, and Repair: In this section, you’ll be tested on your ability to diagnose and adjust wheel alignment. This includes understanding wheel alignment specifications, diagnosing alignment-related issues, and adjusting front and rear wheel alignment to manufacturer’s specifications.
- Wheel and Tire Diagnosis and Service: This section evaluates your understanding of wheel and tire components and their functions. You’ll need to diagnose problems such as abnormal tire wear and vibration, and perform necessary services such as tire balancing, tire rotation, and wheel bearing replacement.
To perform well on the A4 Suspension and Steering Test, you need a solid understanding of suspension and steering components and their functions, as well as practical skills in diagnosing and repairing suspension and steering-related problems. It’s also important to have a thorough understanding of safety procedures and the use of service manuals and other technical literature.
Table: Suspension and Steering (Test A4) Content Areas
Content Area | Number of Questions | Percentage of Test |
---|---|---|
A. Steering Systems Diagnosis and Repair | 12 | 30% |
B. Suspension Systems Diagnosis and Repair | 12 | 30% |
C. Wheel Alignment Diagnosis, Adjustment, and Repair | 11 | 28% |
D. Wheel and Tire Diagnosis and Service | 5 | 12% |
Total | 40 | 100% |
A4 Sample Question
A vehicle pulls to the right when the brakes are applied. The brake system has been inspected and no faults were found. The MOST likely cause of this problem is:
- A worn left front wheel bearing.
- A faulty power steering pump.
- An underinflated right front tire.
- A clogged fuel injector.
Answer: C. An underinflated right front tire.
Explanation:
If a vehicle pulls to one side when the brakes are applied and no faults are found in the brake system, a tire issue is a likely cause. An underinflated tire (Answer C) on the side the vehicle is pulling towards can cause this symptom.
Answer A is incorrect because a worn wheel bearing would typically cause a rumbling noise and possibly play in the wheel, but it wouldn’t typically cause the vehicle to pull to one side when braking.
Answer B is incorrect because a faulty power steering pump would typically cause difficulty in steering or a whining noise, not a pull to one side when braking.
Answer D is incorrect because a clogged fuel injector would typically cause engine performance problems, not a pull to one side when braking.
A5 – Brakes Test
The A5 Brakes Test is part of the ASE Certification series for Master Automobile Technician certification. This test is designed to evaluate a technician’s knowledge and skills in the area of brake systems in automobiles.
The test consists of 55 multiple-choice questions, with an additional 10 research questions that do not count towards the final score. The time allotted for the test is 1 1/4 hours.
The A5 Brakes Test covers several key areas:
- Hydraulic System Diagnosis and Repair: This section assesses your ability to diagnose and repair problems related to hydraulic brake systems. This includes identifying abnormal pedal feel, checking brake fluid condition and level, diagnosing issues with master cylinders and brake boosters, and repairing or replacing hydraulic system components.
- Drum Brake Diagnosis and Repair: This section tests your skills in diagnosing and repairing drum brake problems. This includes diagnosing issues related to brake noise, brake pedal feel, and parking brake operation. It also involves repairing or replacing drum brake components such as shoes, drums, wheel cylinders, and hardware.
- Disc Brake Diagnosis and Repair: In this section, you’ll be tested on your ability to diagnose and repair disc brake problems. This includes diagnosing issues related to brake noise, brake pedal feel, and brake pulsation. It also involves repairing or replacing disc brake components such as pads, rotors, calipers, and hardware.
- Power Assist Units Diagnosis and Repair: This section evaluates your understanding of power brake assist units and their functions. You’ll need to diagnose problems related to brake pedal effort and repair or replace power assist units as necessary.
- Electronic Brake Control Systems (ABS, TCS, ECS) Diagnosis and Repair: This section tests your ability to diagnose and repair problems related to electronic brake control systems. This includes understanding system operation, diagnosing system faults using a scan tool, and repairing or replacing system components.
To perform well on the A5 Brakes Test, you need a solid understanding of brake system components and their functions, as well as practical skills in diagnosing and repairing brake-related problems. It’s also important to have a thorough understanding of safety procedures and the use of service manuals and other technical literature.
Table: Brakes (Test A5) Content Areas
Content Area | Number of Questions | Percentage of Test |
---|---|---|
A. Hydraulic, Power Assist, and Parking Brake Systems Diagnosis and Repair | 19 | 43% |
B. Drum Brake Diagnosis and Repair | 5 | 11% |
C. Disc Brake Diagnosis and Repair | 11 | 24% |
D. Electronic Brake Control Systems: Antilock Brake System (ABS), Traction Control System (TCS), and Electronic Stability Control System (ESC) Diagnosis and Repair | 10 | 22% |
Total | 45 | 100% |
A5 Sample Question
A vehicle’s brake pedal feels spongy and the brake fluid level in the master cylinder is correct. The MOST likely cause of this problem is:
- Air in the brake lines.
- A worn brake pad.
- A faulty power steering pump.
- A clogged fuel filter.
Answer: A. Air in the brake lines.
Explanation:
A spongy brake pedal is often caused by air in the brake lines (Answer A). Air in the hydraulic system of the brakes can compress, unlike the brake fluid, which leads to a spongy or soft brake pedal feel.
Answer B is incorrect because worn brake pads would typically cause a decrease in braking performance or a squealing noise, not a spongy brake pedal.
Answer C is incorrect because a faulty power steering pump would typically cause steering-related problems, not a spongy brake pedal.
Answer D is incorrect because a clogged fuel filter would typically cause engine performance problems, not a spongy brake pedal.
A6 – Electrical/Electronic Systems Test
The A6 Electrical/Electronic Systems Test is part of the ASE Certification series for Master Automobile Technician certification. This test is designed to evaluate a technician’s knowledge and skills in the area of electrical and electronic systems in automobiles.
The test consists of 60 multiple-choice questions, with an additional 10 research questions that do not count towards the final score. The time allotted for the test is 1 1/2 hours.
The A6 Electrical/Electronic Systems Test covers several key areas:
- General Electrical System Diagnosis: This section assesses your ability to diagnose general electrical system problems. This includes using a multimeter to measure voltage, current, and resistance, interpreting wiring diagrams, and diagnosing issues with fuses, circuit breakers, relays, and wiring.
- Battery and Starting System Diagnosis and Repair: This section tests your skills in diagnosing and repairing battery and starting system problems. This includes diagnosing battery condition, testing the starting system for proper operation, and repairing or replacing components as necessary.
- Charging System Diagnosis and Repair: In this section, you’ll be tested on your ability to diagnose and repair charging system problems. This includes testing the charging system for proper operation, diagnosing issues with the alternator and voltage regulator, and repairing or replacing components as necessary.
- Lighting Systems Diagnosis and Repair: This section evaluates your understanding of automotive lighting systems and their functions. You’ll need to diagnose problems with headlights, tail lights, turn signals, and interior lights, and repair or replace components as necessary.
- Instrument Cluster and Driver Information Systems Diagnosis and Repair: This section tests your ability to diagnose and repair problems with instrument clusters and driver information systems. This includes diagnosing issues with gauges, warning lights, and information displays, and repairing or replacing components as necessary.
- Body Electrical Systems Diagnosis and Repair: This section assesses your ability to diagnose and repair body electrical system problems. This includes diagnosing issues with power windows, power seats, power mirrors, and other body electrical systems, and repairing or replacing components as necessary.
To perform well on the A6 Electrical/Electronic Systems Test, you need a solid understanding of electrical and electronic system components and their functions, as well as practical skills in diagnosing and repairing electrical and electronic system-related problems. It’s also important to have a thorough understanding of safety procedures and the use of service manuals and other technical literature.
Table: Electrical/Electronic Systems (Test A6) Content Areas
Content Area | Number of Questions | Percentage of Test |
---|---|---|
A. General Electrical/Electronic System Diagnosis | 13 | 26% |
B. Battery and Starting System Diagnosis and Repair | 9 | 18% |
C. Charging System Diagnosis and Repair | 5 | 10% |
D. Lighting Systems Diagnosis and Repair | 6 | 12% |
E. Instrument Cluster and Driver Information Systems Diagnosis and Repair | 6 | 12% |
F. Body Electrical Systems Diagnosis and Repair | 11 | 22% |
Total | 50 | 100% |
A6 Sample Question
A vehicle’s battery drains overnight. All lights are turned off and no obvious electrical devices are left on. The MOST likely cause of this problem is:
- A faulty alternator.
- A parasitic draw.
- A worn serpentine belt.
- A clogged fuel filter.
Answer: B. A parasitic draw.
Explanation:
If a vehicle’s battery drains overnight with no obvious electrical devices left on, the most likely cause is a parasitic draw (Answer B). This is when an electrical device continues to use power from the battery even after the vehicle is turned off.
Answer A is incorrect because a faulty alternator would typically cause the battery to drain while the vehicle is running, not when it’s turned off.
Answer C is incorrect because a worn serpentine belt would typically cause issues with the operation of the alternator, power steering pump, or air conditioning compressor, but it wouldn’t cause the battery to drain overnight.
Answer D is incorrect because a clogged fuel filter would typically cause engine performance problems, not a battery drain.
A7 – Heating and Air Conditioning Test
The A7 Heating and Air Conditioning Test is part of the ASE Certification series for Master Automobile Technician certification. This test is designed to evaluate a technician’s knowledge and skills in the area of heating and air conditioning systems in automobiles.
The test consists of 60 multiple-choice questions, with an additional 10 research questions that do not count towards the final score. The time allotted for the test is 1 1/4 hours.
The A7 Heating and Air Conditioning Test covers several key areas:
- A/C System Diagnosis and Repair: This section assesses your ability to diagnose and repair air conditioning system problems. This includes understanding system operation, diagnosing issues with system performance, and repairing or replacing A/C system components.
- Refrigeration System Component Diagnosis and Repair: This section tests your skills in diagnosing and repairing refrigeration system component problems. This includes diagnosing issues with the compressor, condenser, evaporator, metering devices, and refrigerant lines, and repairing or replacing components as necessary.
- Heating and Engine Cooling Systems Diagnosis and Repair: In this section, you’ll be tested on your ability to diagnose and repair heating and engine cooling system problems. This includes diagnosing issues with the heater, radiator, water pump, thermostat, and cooling fans, and repairing or replacing components as necessary.
- Operating Systems and Related Controls Diagnosis and Repair: This section evaluates your understanding of heating and air conditioning operating systems and their related controls. You’ll need to diagnose problems with the control panel, blower motor, temperature doors, mode doors, and sensors, and repair or replace components as necessary.
To perform well on the A7 Heating and Air Conditioning Test, you need a solid understanding of heating and air conditioning system components and their functions, as well as practical skills in diagnosing and repairing heating and air conditioning system-related problems. It’s also important to have a thorough understanding of safety procedures and the use of service manuals and other technical literature.
Table: Heating and Air Conditioning (Test A7) Content Areas
Content Area | Number of Questions | Percentage of Test |
---|---|---|
A. Heating, Ventilation, A/C (HVAC) and Engine Cooling System Service, Diagnosis, and Repair | 21 | 42% |
B. Refrigeration System Component Diagnosis and Repair | 10 | 20% |
C. Operating Systems and Related Controls Diagnosis and Repair | 19 | 38% |
Total | 50 | 100% |
A7 Sample Question
A vehicle’s air conditioning system is not cooling effectively. The compressor is engaging and the system is properly charged with refrigerant. The MOST likely cause of this problem is:
- A faulty thermostat.
- A clogged condenser.
- A worn serpentine belt.
- A clogged fuel filter.
Answer: B. A clogged condenser.
Explanation:
If a vehicle’s air conditioning system is not cooling effectively, but the compressor is engaging and the system is properly charged, the most likely cause is a clogged condenser (Answer B). The condenser’s job is to expel heat from the refrigerant. If it’s clogged, it can’t effectively cool the refrigerant, which leads to poor cooling performance.
Answer A is incorrect because a faulty thermostat would typically cause engine temperature problems, not issues with the air conditioning system.
Answer C is incorrect because a worn serpentine belt could cause the compressor to not operate effectively, but in this case, the compressor is engaging properly.
Answer D is incorrect because a clogged fuel filter would typically cause engine performance problems, not issues with the air conditioning system.
A8 – Engine Performance Test
The A8 Engine Performance Test is part of the ASE Certification series for Master Automobile Technician certification. This test is designed to evaluate a technician’s knowledge and skills in the area of engine performance systems in automobiles.
The test consists of 60 multiple-choice questions, with an additional 10 research questions that do not count towards the final score. The time allotted for the test is 1 1/4 hours.
The A8 Engine Performance Test covers several key areas:
- General Engine Performance Diagnosis: This section assesses your ability to diagnose general engine performance problems. This includes understanding engine operation, diagnosing issues with engine performance, and interpreting scan tool data.
- Ignition System Diagnosis and Repair: This section tests your skills in diagnosing and repairing ignition system problems. This includes diagnosing issues with spark plugs, ignition coils, distributor caps and rotors, and ignition control modules, and repairing or replacing components as necessary.
- Fuel, Air Induction, and Exhaust Systems Diagnosis and Repair: In this section, you’ll be tested on your ability to diagnose and repair fuel, air induction, and exhaust system problems. This includes diagnosing issues with fuel pumps, fuel injectors, air filters, intake manifolds, exhaust manifolds, and catalytic converters, and repairing or replacing components as necessary.
- Emissions Control Systems Diagnosis and Repair: This section evaluates your understanding of emissions control systems and their functions. You’ll need to diagnose problems with the PCV system, EGR system, EVAP system, and catalytic converter, and repair or replace components as necessary.
- Computerized Engine Controls Diagnosis and Repair: This section tests your ability to diagnose and repair problems with computerized engine controls. This includes understanding system operation, diagnosing issues with sensors and actuators, and interpreting scan tool data.
To perform well on the A8 Engine Performance Test, you need a solid understanding of engine performance system components and their functions, as well as practical skills in diagnosing and repairing engine performance-related problems. It’s also important to have a thorough understanding of safety procedures and the use of service manuals and other technical literature.
Table: Engine Performance (Test A8) Content Areas
Content Area | Number of Questions | Percentage of Test |
---|---|---|
A. General Diagnosis | 12 | 24% |
B. Ignition System Diagnosis and Repair | 8 | 16% |
C. Fuel, Air Induction and Exhaust Systems Diagnosis and Repair | 9 | 18% |
D. Emissions Control Systems Diagnosis and Repair | 8 | 16% |
E. Computerized Engine Controls Diagnosis and Repair | 13 | 26% |
Total | 50 | 100% |
A8 Sample Question
A vehicle has poor fuel economy and the check engine light is on. The scan tool shows a code for a lean condition. The MOST likely cause of this problem is:
- A faulty spark plug.
- A clogged fuel injector.
- A worn serpentine belt.
- A leaking intake manifold gasket.
Answer: D. A leaking intake manifold gasket.
Explanation:
A lean condition means there is too much air and not enough fuel in the air-fuel mixture. This can be caused by a leak in the intake manifold gasket (Answer D), which allows extra air to enter the engine. This can lead to poor fuel economy and trigger the check engine light.
Answer A is incorrect because a faulty spark plug would typically cause a misfire or rough idle, not a lean condition.
Answer B is incorrect because a clogged fuel injector can cause a lean condition, but it would typically also cause a misfire or rough idle, not just poor fuel economy and a check engine light.
Answer C is incorrect because a worn serpentine belt would typically cause issues with the operation of the alternator, power steering pump, or air conditioning compressor, but it wouldn’t cause a lean condition.
Preparation Strategies
Preparing for the ASE Certification Test is a significant undertaking that requires a strategic approach. As an aspiring Master Automobile Technician, it’s crucial to understand that this is not just a test of memory, but a measure of your understanding of complex automotive systems and your ability to apply this knowledge in real-world scenarios. Here are some proven strategies to help you prepare effectively for the ASE Certification Test:
- Understand the Test Structure: Familiarize yourself with the structure of the ASE Certification Test. Understand the different sections (A1-A8) and what each one covers. This will help you know what to expect and how to allocate your study time effectively.
- Review the Official ASE Study Guides: ASE provides comprehensive study guides for each test series. These guides outline the information that will be covered in the test, provide sample questions, and explain the reasoning behind the correct answers.
- Hands-On Practice: There’s no substitute for hands-on experience. The ASE tests are designed to measure your ability to apply knowledge in real-world scenarios. Spend time in a workshop, if possible, to get practical experience with the systems and components you’ll be tested on.
- Study Regularly and Consistently: Cramming is not an effective strategy for the ASE Certification Test. Instead, set up a regular study schedule that allows you to cover all the material over several weeks or months. This will help you retain the information better and reduce stress as the test date approaches.
- Use Quality Study Materials: While there are many study materials available, not all are created equal. Look for resources that are up-to-date, comprehensive, and aligned with the ASE test objectives.
- Practice with Sample Questions: Practice tests are a great way to check your understanding and get a feel for the types of questions you’ll encounter on the actual test. They can also help you identify areas where you need to focus your study efforts.
- Take Care of Your Physical Health: Good physical health supports mental acuity. Make sure to get regular exercise, eat a healthy diet, and get plenty of sleep in the weeks leading up to the test. This will help you be at your best on test day.
- Stay Positive and Confident: A positive mindset can significantly impact your test performance. Remember, you’re studying to become a certified expert in your field. Stay confident in your abilities and keep a positive attitude throughout your preparation and during the test.
Preparing for the ASE Certification Test is a journey that requires dedication, discipline, and a strategic approach. By following these strategies, you’ll be well on your way to achieving a successful outcome. Remember, the goal is not just to pass the test, but to truly understand and be able to apply the knowledge and skills being tested. This is what will make you a valuable asset in the automotive industry.
Here is my study pattern. First I take sample tests and mark any questions that I get wrong or am not certain about. Then I review these identified weak areas. Then I test again. I switch topic areas so that the information sinks in. Then I return to make certain that I remember the material. Rinse and repeat until your test date arrives! Get a good night’s sleep before your appointment at the test center.
Source: Predictable-Past-912/reddit
Test Features
Purpose of the ASE Certification Test
The ASE Certification Test is designed to measure the skills and knowledge of automotive service professionals. The test’s primary purpose is to certify that these professionals have the technical competence required to perform their job at a high level. By passing the ASE Certification Test, technicians demonstrate to employers, customers, and themselves that they have the expertise needed to excel in their field.
Structure of the ASE Certification Test
The ASE Certification Test is divided into several sections, each focusing on a different aspect of automotive service. For the Master Automobile Technician certification, these sections are A1 (Engine Repair), A2 (Automatic Transmission/Transaxle), A3 (Manual Drive Train and Axles), A4 (Suspension and Steering), A5 (Brakes), A6 (Electrical/Electronic Systems), A7 (Heating and Air Conditioning), and A8 (Engine Performance). Each section consists of multiple-choice questions that test both theoretical knowledge and practical skills.
Scoring of the ASE Certification Test
The ASE Certification Test uses a scaled scoring system. While the number of questions varies between sections, each test’s passing score is set at a scaled score of 70. This does not mean that you need to answer 70% of the questions correctly to pass. Instead, the raw score (the number of questions answered correctly) is converted into a scaled score that takes into account the difficulty of the questions.
Test Administration
The ASE Certification Test is administered at Prometric test centers across the United States. The test is computer-based, and the questions are multiple-choice. Test-takers must register and schedule their test in advance.
Test Preparation
Preparing for the ASE Certification Test requires a combination of theoretical study and hands-on practice. ASE provides study guides for each test section, and many other resources are available. Practical experience, either on the job or through a training program, is also crucial for success on the test.
Recognition and Benefits
Achieving ASE Certification is a significant accomplishment that is recognized throughout the automotive service industry. Certified technicians often have better job opportunities, higher earning potential, and increased job satisfaction. They also demonstrate to customers that they are committed to providing high-quality service.
Common names for the ASE A1-A8 Certification Test
The ASE Master Automobile Technician Test is commonly referred to by several names:
- ASE Master Technician Test
- ASE Master Certification Test
- ASE A-Series Test
- ASE Master Auto Tech Test
- ASE Master Automobile Service Test
- ASE Master Mechanic Test
- ASE Master Auto Repair Test
- ASE Master Certification Exam
- ASE Master Automobile Technician Certification Exam
Technical Facts
Test Fast Facts (tl;dr)
- Computer-based, multiple-choice.
- Eight tests (A1-A8) for certification.
- 50-60 scored questions per test.
- Test duration: 1 to 1.5 hours.
- Passing score: scaled 70.
- Administered at Prometric centers.
- Four testing windows yearly.
- Two years on-the-job training required.
- Retake after 30 days if failed.
- Results emailed post-test.
Test Format and Structure
The ASE Certification Test is a computer-based test that uses a multiple-choice format. Each of the A1-A8 tests for the Master Automobile Technician certification focuses on a different area of automotive service and repair, including engine repair, automatic transmission/transaxle, manual drive train and axles, suspension and steering, brakes, electrical/electronic systems, heating and air conditioning, and engine performance.
Number of Questions and Test Duration
Each test in the A1-A8 series has a different number of questions, ranging from 50 to 60. In addition, each test includes 10 research questions that do not count towards the final score. These research questions are used to gather data for future tests and are indistinguishable from the scored questions. The time allotted for each test in the A1-A8 series varies, ranging from 1 hour to 1 1/2 hours. This time includes not only the time to answer the scored questions but also the research questions.
Scoring System
The ASE Certification Test uses a scaled scoring system. Each test’s passing score is set at a scaled score of 70. The raw score (the number of questions answered correctly) is converted into a scaled score that takes into account the difficulty of the questions. This means that the number of questions you need to answer correctly to pass can vary depending on the specific questions on your test.
Test Administration and Frequency
The ASE Certification Test is administered at Prometric test centers across the United States. The test is offered throughout the year during four testing windows, each lasting two months with a one-month break in between.
Eligibility Requirements
To be eligible to take the ASE Certification Test, candidates must have at least two years of on-the-job training or one year of on-the-job training and a two-year degree in automotive repair. This ensures that candidates have the practical experience necessary to apply the knowledge tested in the exam.
Yep, ASE requires a minimum of two years of actual on-the-job experience to receive certs, and they do call shops to confirm. You can take and pass the tests before/without the experience but they won’t certify without it.
Source: WebMaka/reddit
Retake Policy
If a candidate fails an ASE Certification Test, they can retake the test after a 30-day waiting period. There is no limit to the number of times a candidate can retake the test, allowing for continuous learning and improvement.
Recognition
ASE Certification is recognized nationwide as a mark of excellence in automotive service and repair. Certified technicians are often preferred by employers and customers, and they may have better job opportunities and higher earning potential.
Test Preparation and Registration
Registration for the ASE Certification Test can be done online at the official ASE website. The website also provides access to electronic versions of all Official ASE Study Guides. Creating a personal myASE account allows candidates to register for tests, make test appointments, and access their personal ASE certification information.
Test Results
Test results are emailed to candidates immediately following their testing appointment. Results will not be given over the phone nor will they be released to anyone without the candidate’s written permission. If a candidate fails a test, they must wait 30 days from the test date before they can purchase it again.
The table below provides information about the administration and eligibility for the ASE Certification Test, including the test location, schedule, eligibility requirements, and retake policy.
Aspect | Details |
---|---|
Test Location | Administered at Prometric test centers across the U.S. |
Test Schedule | Offered during four testing windows each year |
Eligibility | Requires at least two years of on-the-job training or equivalent |
Retake Policy | Can retake after a 30-day waiting period if failed |
Results Scale and Interpretations
The ASE Certification Test uses a scaled scoring system. Each test’s passing score is set at a scaled score of 70. The raw score, which is the number of questions answered correctly, is converted into a scaled score. This conversion takes into account the difficulty of the questions, meaning that the number of questions you need to answer correctly to pass can vary depending on the specific questions on your test.
The table below provides details about the format and scoring of the ASE Certification Test, including the number of questions, scoring system, and test duration.
Aspect | Details |
---|---|
Test Format | Computer-based, multiple-choice |
Number of Questions | 50-60 scored questions per test |
Scoring System | Scaled scoring system, passing score is 70 |
Test Duration | 1 to 1.5 hours |
Here’s a detailed breakdown of the different components of the score report:
- Raw Score: This is the number of questions you answered correctly. Each ASE test has a different number of questions, ranging from 40 to 75. However, not all questions are scored. Some are used for research purposes only.
- Scaled Score: Your raw score is converted into a scaled score, which ranges from 0 to 100. The conversion process takes into account the difficulty of the questions on your test. A scaled score of 70 is required to pass the test.
- Percentile Ranking: This indicates the percentage of test-takers who scored lower than you. For example, if your percentile ranking is 80, it means that you scored higher than 80% of test-takers.
- Sub-Scores: Some tests provide sub-scores, which show your performance in different content areas of the test. These can help you identify your strengths and areas for improvement.
- Score Range: This shows the range of scores that are possible on the test. It can give you an idea of how much room there is for improvement.
- Pass/Fail Status: This simply indicates whether you passed or failed the test.
Here’s an example of how to interpret a score report:
Let’s say you took the A5 Brakes test, which has 45 scored questions. If you answered 35 questions correctly, your raw score would be 35. This raw score would then be converted into a scaled score. If the scaled score is 75, it means you passed the test because the passing score is 70. If your percentile ranking is 85, it means you scored higher than 85% of test-takers. If the test provides sub-scores, you might see that you did particularly well in the area of “Hydraulic Systems,” but need to improve in “Drum Brake Systems.”
Your score report can be used to assess your ability to perform in the areas tested by the ASE Certification Test. If you pass, it validates your knowledge and skills in those areas. If you don’t pass, you can use the information in the score report to focus your study efforts for a retake. For example, you might spend more time studying areas where your sub-scores were low.
Remember, if you fail a test, you must wait 30 days from the test date before you can purchase it again. But don’t be discouraged. Use that time to prepare and improve, and remember that many successful technicians didn’t pass on their first attempt.
iPREP: Concise. Focused. What you need.
Sign up
Immediate access
Practice
Online self-paced
Pass
Ace that Test!
ASE Certification FAQs
The ASE Certification Test is a professional certification exam administered by the National Institute for Automotive Service Excellence. It is designed to validate the technical knowledge and skills of individuals in various areas of automotive service and repair.
The ASE Certification Test measures a range of skills and knowledge related to automotive service and repair, including engine repair, automatic transmission/transaxle, manual drive train and axles, suspension and steering, brakes, electrical/electronic systems, heating and air conditioning, and engine performance.
The ASE Certification Test is a computer-based test that uses a multiple-choice format. Each test in the A1-A8 series focuses on a different area of automotive service and repair.
The ASE Certification Test uses a scaled scoring system. Each test’s passing score is set at a scaled score of 70. The raw score, which is the number of questions answered correctly, is converted into a scaled score that takes into account the difficulty of the questions.
ASE Certification is recognized nationwide as a mark of excellence in automotive service and repair. Certified technicians are often preferred by employers and customers, and they may have better job opportunities and higher earning potential.
The ASE Certification Test is offered throughout the year during four testing windows, each lasting two months with a one-month break in between. If a candidate fails a test, they can retake the test after a 30-day waiting period.
To be eligible to take the ASE Certification Test, candidates must have at least two years of on-the-job training or one year of on-the-job training and a two-year degree in automotive repair.
On the day of the test, candidates should bring their Admission Ticket and an unexpired, government-issued photo ID. Books, notes, calculators, and electronic devices (including cell phones) are not permitted in the test room.
Registration for the ASE Certification Test can be done online at the official ASE website. The website also provides access to electronic versions of all Official ASE Study Guides. Creating a personal myASE account allows candidates to register for tests, make test appointments, and access their personal ASE certification information.
Test results are emailed to candidates immediately following their testing appointment. Results will not be given over the phone nor will they be released to anyone without the candidate’s written permission.
Test Tips
Here are seven practical and effective test-taking tips for those about to take the ASE Certification Test:
- Arrive Early: On the day of the test, make sure to arrive at the test center well in advance. This will give you ample time to locate the test center, park, and settle in without feeling rushed or stressed.
- Bring Necessary Documents: Don’t forget to bring your Admission Ticket and an unexpired, government-issued photo ID. Without these, you won’t be allowed to take the test.
- Leave Personal Items at Home: Books, notes, calculators, and electronic devices (including cell phones) are not permitted in the test room. You will be assigned a locker to store personal items, but it’s best to bring only what’s necessary to avoid any potential issues.
- Read Questions Carefully: Each question on the ASE Certification Test is designed to test a specific skill or piece of knowledge. Make sure to read each question carefully to understand exactly what it’s asking before you choose your answer.
- Use the Process of Elimination: If you’re unsure about a question, try to eliminate the incorrect answers first. This can often make the correct answer more apparent and can be a useful strategy when you’re not sure.
- Flag Difficult Questions: If a question is particularly challenging, don’t spend too much time on it. Flag it for review and move on to the next question. You can come back to it later if time allows.
- Answer Every Question: Since the score is based on the number of correct answers, it’s in your best interest to answer every question. Even if you’re not sure, it’s better to make an educated guess than to leave a question blank.
Remember, the ASE Certification Test is a professional exam that requires both knowledge and strategy. By following these tips, you’ll be well-prepared to tackle the test and demonstrate your automotive service expertise.
Administration
- Test Location: The ASE Certification Test is administered at Prometric test centers across the United States.
- Test Schedule: The test is offered throughout the year during four testing windows, each lasting two months with a one-month break in between.
- Test Format: The ASE Certification Test is a computer-based test that uses a multiple-choice format. Each test in the A1-A8 series focuses on a different area of automotive service and repair.
- Test Materials: Candidates are not allowed to bring any personal items into the test room, including books, notes, calculators, and electronic devices (including cell phones). Pen and paper are not needed as the test is computer-based.
- Cost: The registration fee for the ASE Certification Test is $36, and each test costs an additional $47. The advanced level tests (L1, L2, and L3) cost $94 each.
- Retake Policy: If a candidate fails an ASE Certification Test, they can retake the test after a 30-day waiting period. There is no limit to the number of times a candidate can retake the test.
Test Provider
The National Institute for Automotive Service Excellence (ASE), established in 1972, is a non-profit organization that aims to improve the quality of vehicle repair and service. It does this by testing and certifying automotive professionals, with nearly 400,000 certified professionals to date. The ASE program has industry-wide acceptance and recognition and is national in scope.
The ASE offers a series of exams for professionals in the automotive repair and service industry, grouped into specialties such as automobile, medium/heavy truck, and more. The most recognized certification is the ASE Master Technician certification. The ASE conducts its testing through Prometric Test Centers, which are located throughout the United States.
Information Sources
- National Institute for Automotive Service Excellence
- Prometric Test Centers
- Official ASE Education Foundation
Disclaimer – All the information and prep materials on iPREP are genuine and were created for tutoring purposes. iPREP is not affiliated with the National Institute for Automotive Service Excellence (ASE), which is the owner of the ASE Certification Test, or with any of the companies mentioned above.
Free ASE practice test: Get to know what the ASE Certification Test will be like by practicing with these sample questions:
A1 – Engine Repair Sample Questions
A1 – Question 1 of 5
A customer brings in their car complaining of a rough idle. You notice that the Check Engine Light is on and retrieve a P0302 code. What does this code indicate?
- A problem with the fuel injector
- A misfire in cylinder 3
- A problem with the oxygen sensor
- A misfire in cylinder 2
The correct answer is (D). A P0302 code indicates a misfire in cylinder 2.
Explanation:
- Option (B) is incorrect because a P0303 code would indicate a misfire in cylinder 3.
- Option (C) is incorrect because a problem with the oxygen sensor would trigger a different code, typically starting with P013 or P014.
- Option (A) is incorrect because a problem with the fuel injector would also trigger a different code, typically starting with P02 or P03. However, it’s worth noting that a faulty fuel injector could potentially cause a misfire in a specific cylinder, which might lead to a P0302 code. But the code itself specifically indicates a misfire, not a fuel injector issue.
A1 – Question 2 of 5
A customer’s vehicle has an oil leak. After inspecting the vehicle, you determine that the leak is coming from the area around the oil filter. What is the most likely cause of this leak?
- The oil filter is not tightened properly.
- The oil pan gasket is leaking.
- The valve cover gasket is leaking.
- The head gasket is leaking.
The correct answer is (A). The oil filter is not tightened properly.
Explanation:
- Option (B) is incorrect because an oil pan gasket leak would typically result in oil leaking from the bottom of the engine, not the area around the oil filter.
- Option (C) is incorrect because a valve cover gasket leak would typically result in oil leaking from the top of the engine, not the area around the oil filter.
- Option (D) is incorrect because a head gasket leak could cause oil to mix with coolant, result in white smoke from the exhaust, or cause overheating, but it would not typically cause a leak around the oil filter.
A1 – Question 3 of 5
A customer complains of a squealing noise from the engine compartment that gets worse when the air conditioning is turned on. What is the most likely cause of this noise?
- The air conditioning compressor is failing.
- The serpentine belt is worn or loose.
- The power steering pump is failing.
- The timing belt is worn or loose.
The correct answer is (B). The serpentine belt is worn or loose.
Explanation:
- Option (A) is incorrect because a failing air conditioning compressor would typically make a grinding or rattling noise, not a squealing noise.
- Option (C) is incorrect because a failing power steering pump would typically make a whining noise, especially when turning the steering wheel, not a squealing noise.
- Option (D) is incorrect because a worn or loose timing belt would typically result in a ticking noise coming from inside the engine, not a squealing noise from the engine compartment.
A1 – Question 4 of 5
A customer complains that their car is hard to start in the morning, but once it starts, it runs fine for the rest of the day. What is the most likely cause of this problem?
- The battery is weak.
- The fuel pump is failing.
- The spark plugs are worn out.
- The engine coolant temperature sensor is faulty.
The correct answer is (D). The engine coolant temperature sensor is faulty.
Explanation:
- Option (A) is incorrect because a weak battery would typically cause the car to have trouble starting all the time, not just in the morning.
- Option (B) is incorrect because a failing fuel pump would typically cause the car to have performance issues even after it starts, such as hesitation or stalling.
- Option (C) is incorrect because worn out spark plugs could cause a variety of issues, including hard starting, but these issues would typically not be limited to the morning.
- Option (D) is correct because the engine coolant temperature sensor plays a key role in the engine’s cold start enrichment. If the sensor is faulty and reads the engine temperature as warm when it’s actually cold, the engine control unit (ECU) may not provide enough fuel for a cold start, causing the hard starting issue in the morning. Once the engine is running and warms up, this issue would not affect drivability.
A1 – Question 5 of 5
Based on the diagram, which of the following is most likely to cause a loss of compression in this cylinder?
- A worn out connecting rod bearing.
- A broken piston ring.
- A worn out crankshaft bearing.
- A loose spark plug.
The correct answer is (B). A broken piston ring.
Explanation:
- Option (A) is incorrect because a worn out connecting rod bearing would cause a knocking noise and potentially engine damage, but it would not typically cause a loss of compression.
- Option (C) is incorrect because a worn out crankshaft bearing would cause a knocking noise and potentially engine damage, but it would not typically cause a loss of compression.
- Option (D) is incorrect because a loose spark plug could cause a misfire and a loss of power, but it would not typically cause a loss of compression.
- Option (B) is correct because a broken piston ring can cause a loss of compression. The piston rings seal the gap between the piston and the cylinder wall. If a ring is broken, compression can escape past the ring, leading to a loss of compression in the cylinder.
A2 – Automatic Transmission/Transaxle Sample Questions
A2 – Question 1 of 5
A customer complains that their vehicle’s automatic transmission is slipping and the fluid is dark and smells burnt. What is the most likely cause of these symptoms?
- The transmission fluid level is low.
- The transmission fluid needs to be changed.
- The transmission filter is clogged.
- The transmission is overheating.
The correct answer is (D). The transmission is overheating.
Explanation:
- Option (A) is incorrect because while a low transmission fluid level can cause slipping, it would not typically cause the fluid to become dark and smell burnt.
- Option (B) is incorrect because while old transmission fluid can become dark and smell burnt, it would not typically cause the transmission to slip unless it has degraded to the point of causing overheating or other damage.
- Option (C) is incorrect because a clogged transmission filter can cause slipping and other performance issues, but it would not typically cause the fluid to become dark and smell burnt unless it has led to overheating or other damage.
- Option (D) is correct because overheating can cause both slipping and the fluid to become dark and smell burnt. Overheating can be caused by several factors, including low fluid level, old or degraded fluid, a clogged filter, or mechanical issues with the transmission.
A2 – Question 2 of 5
A customer’s vehicle is experiencing delayed shifting and the Check Engine Light is on. You retrieve a P0753 code. What does this code indicate?
- Torque converter clutch solenoid circuit electrical.
- Transmission fluid temperature sensor circuit high input.
- Pressure control solenoid A control circuit range/performance.
- Shift solenoid A electrical fault.
The correct answer is (D). Shift solenoid A electrical fault.
Explanation:
- Option (A) is incorrect because a torque converter clutch solenoid circuit electrical would trigger a different code, typically starting with P074.
- Option (B) is incorrect because a transmission fluid temperature sensor circuit high input would trigger a different code, typically starting with P071.
- Option (C) is incorrect because a pressure control solenoid A control circuit range/performance would trigger a different code, typically starting with P079.
- Option (D) is correct because a P0753 code specifically indicates a shift solenoid A electrical fault. This could cause delayed shifting as the solenoid controls the flow of fluid in the transmission and helps to engage the appropriate gear.
A2 – Question 3 of 5
A customer complains that their vehicle’s automatic transmission is not shifting into overdrive. You notice that the overdrive light is flashing. What is the most likely cause of this problem?
- The overdrive solenoid is faulty.
- The transmission fluid level is low.
- The vehicle speed sensor is faulty.
- The transmission control module is faulty.
The correct answer is (A). The overdrive solenoid is faulty.
Explanation:
- Option (B) is incorrect because while a low transmission fluid level can cause a variety of transmission issues, it would not typically cause the overdrive light to flash.
- Option (C) is incorrect because a faulty vehicle speed sensor could cause shifting issues and other performance problems, but it would not typically cause the overdrive light to flash.
- Option (D) is incorrect because while a faulty transmission control module could cause a variety of transmission issues, including problems with overdrive, it would typically cause other symptoms as well, such as shifting issues in other gears or a failure to shift at all.
- Option (A) is correct because a faulty overdrive solenoid could prevent the transmission from shifting into overdrive and cause the overdrive light to flash. The overdrive solenoid controls the flow of fluid to the overdrive clutch, and if it’s not working properly, the clutch may not engage as it should.
A2 – Question 4 of 5
A customer complains that their vehicle’s automatic transmission is not shifting out of first gear. The fluid level is correct and the fluid is clean. What is the most likely cause of this problem?
- The shift linkage is misadjusted.
- The transmission control module is in “limp mode.”
- The transmission fluid is too thin.
- The torque converter is failing.
The correct answer is (B). The transmission control module is in “limp mode.”
Explanation:
- Option (A) is incorrect because a misadjusted shift linkage would typically cause problems with manual shifting, not automatic shifting.
- Option (C) is incorrect because thin transmission fluid could cause slipping or overheating, but it would not typically prevent the transmission from shifting out of first gear.
- Option (D) is incorrect because a failing torque converter could cause a variety of transmission issues, but it would not typically prevent the transmission from shifting out of first gear.
- Option (B) is correct because when the transmission control module detects a problem that could cause damage to the transmission, it can put the transmission into “limp mode,” which typically limits the transmission to only first gear to prevent further damage.
A2 – Question 5 of 5
Based on the diagram, which of the following is most likely to cause the transmission to slip when shifting from first to second gear?
- A worn out torque converter.
- A faulty fluid pump.
- A worn out clutch pack for second gear.
- A broken tooth on the planetary gear set.
The correct answer is (C). A worn out clutch pack for second gear.
Explanation:
- Option (A) is incorrect because a worn out torque converter would cause a lack of power or a shuddering sensation, but it would not typically cause the transmission to slip when shifting.
- Option (B) is incorrect because a faulty fluid pump could cause a variety of transmission problems, but it would not typically cause the transmission to slip when shifting.
- Option (D) is incorrect because a broken tooth on the planetary gear set could cause a variety of transmission problems, but it would not typically cause the transmission to slip when shifting.
- Option (C) is correct because a worn out clutch pack for second gear can cause the transmission to slip when shifting from first to second gear. The clutch packs are responsible for engaging and disengaging the gears. If the clutch pack for second gear is worn out, it may not fully engage when the transmission shifts into second gear, causing the transmission to slip.
A3 – Manual Drive Train and Axles Sample Questions
A3 – Question 1 of 5
A customer complains of a grinding noise from the rear of their rear-wheel-drive vehicle when making turns. The noise is not present when driving straight. What is the most likely cause of this noise?
- The rear differential fluid is low or contaminated.
- The rear wheel bearings are worn.
- The rear brake pads are worn.
- The rear drive shaft U-joints are worn.
The correct answer is (A). The rear differential fluid is low or contaminated.
Explanation:
- Option (B) is incorrect because worn wheel bearings would typically cause a humming or growling noise that gets louder with speed, not a grinding noise when turning.
- Option (C) is incorrect because worn brake pads would typically cause a squealing or grinding noise when braking, not when turning.
- Option (D) is incorrect because worn U-joints would typically cause a clunking noise when accelerating or decelerating, not a grinding noise when turning.
- Option (A) is correct because the rear differential allows the rear wheels to rotate at different speeds when turning. If the fluid is low or contaminated, it can cause a grinding noise when the wheels are rotating at different speeds, such as when making turns.
A3 – Question 2 of 5
A customer complains that their vehicle’s clutch pedal feels soft and the vehicle is hard to shift. The clutch fluid level is correct and the fluid is clean. What is the most likely cause of this problem?
- The clutch disc is worn.
- The clutch master cylinder is failing.
- The transmission synchronizers are worn.
- The clutch cable is stretched.
The correct answer is (B). The clutch master cylinder is failing.
Explanation:
- Option (A) is incorrect because a worn clutch disc would typically cause slipping, not a soft pedal feel or difficulty shifting.
- Option (C) is incorrect because worn transmission synchronizers would typically cause grinding when shifting, not a soft pedal feel.
- Option (D) is incorrect because a stretched clutch cable would typically cause a high clutch pedal engagement point, not a soft pedal feel.
- Option (B) is correct because the clutch master cylinder generates the hydraulic pressure needed to disengage the clutch when the pedal is pressed. If it’s failing, it may not generate enough pressure, resulting in a soft pedal feel and difficulty shifting.
A3 – Question 3 of 5
A customer complains that their vehicle’s manual transmission pops out of first gear into neutral on its own. The transmission fluid level is correct and the fluid is clean. What is the most likely cause of this problem?
- The first gear synchronizer is worn.
- The clutch disc is worn.
- The transmission mount is broken.
- The shift linkage is misadjusted.
The correct answer is (A). The first gear synchronizer is worn.
Explanation:
- Option (B) is incorrect because a worn clutch disc would typically cause slipping, not the transmission popping out of gear.
- Option (C) is incorrect because a broken transmission mount could cause a variety of issues, including noise and vibration, but it would not typically cause the transmission to pop out of gear.
- Option (D) is incorrect because a misadjusted shift linkage could cause difficulty getting into gear, but it would not typically cause the transmission to pop out of gear.
- Option (A) is correct because the synchronizers in a manual transmission help the gears to mesh smoothly. If a synchronizer is worn, it may not hold the gears together properly, causing the transmission to pop out of gear.
A3 – Question 4 of 5
A customer complains that their vehicle’s manual transmission is hard to shift into all gears. The clutch pedal feels normal and the vehicle does not grind gears when shifting. What is the most likely cause of this problem?
- The clutch disc is worn.
- The transmission fluid is low or contaminated.
- The shift linkage is misadjusted.
- The transmission mount is broken.
The correct answer is (C). The shift linkage is misadjusted.
Explanation:
- Option (A) is incorrect because a worn clutch disc would typically cause slipping or difficulty disengaging the clutch, not difficulty shifting into all gears.
- Option (B) is incorrect because low or contaminated transmission fluid would typically cause noisy operation or difficulty shifting into specific gears, not difficulty shifting into all gears.
- Option (D) is incorrect because a broken transmission mount could cause a variety of issues, including noise and vibration, but it would not typically cause difficulty shifting into all gears.
- Option (C) is correct because the shift linkage connects the gear shift to the transmission. If it’s misadjusted, it can cause difficulty shifting into all gears. The fact that the problem occurs with all gears and there are no other symptoms like grinding or a strange pedal feel suggests a problem with the linkage rather than the transmission or clutch itself.
A3 – Question 5 of 5
Based on the diagram, which of the following is most likely to cause the transmission to grind when shifting from third to fourth gear?
- A worn out input shaft bearing.
- A faulty shift fork for fourth gear.
- A worn out synchronizer for fourth gear.
- A broken tooth on the third gear.
The correct answer is (C). A worn out synchronizer for fourth gear.
Explanation:
- Option (A) is incorrect because a worn out input shaft bearing would cause a whining or growling noise, but it would not typically cause the transmission to grind when shifting.
- Option (B) is incorrect because a faulty shift fork could cause difficulty shifting into or out of a particular gear, but it would not typically cause the transmission to grind when shifting.
- Option (D) is incorrect because a broken tooth on the third gear could cause a variety of transmission problems, but it would not typically cause the transmission to grind when shifting.
- Option (C) is correct because a worn out synchronizer for fourth gear can cause the transmission to grind when shifting from third to fourth gear. The synchronizers are responsible for matching the speed of the gears before they engage. If the synchronizer for fourth gear is worn out, it may not fully synchronize the gears when the transmission shifts into fourth gear, causing the transmission to grind.
A4 – Suspension and Steering Sample Questions
A4 – Question 1 of 5
A customer complains that their vehicle pulls to the right when braking. The brakes do not make any noise and the brake pedal feels normal. What is the most likely cause of this problem?
- The left front brake pads are worn.
- The brake fluid is contaminated.
- The vehicle’s alignment is off.
- The right front brake caliper is sticking.
The correct answer is (D). The right front brake caliper is sticking.
Explanation:
- Option (A) is incorrect because worn brake pads on one side could cause a pull to the opposite side when braking, but there would typically be other symptoms as well, such as noise or a low brake pedal.
- Option (B) is incorrect because contaminated brake fluid would typically cause a spongy brake pedal feel or reduced braking performance, not a pull to one side when braking.
- Option (C) is incorrect because while an off alignment can cause a vehicle to pull to one side, it would typically do so all the time, not just when braking.
- Option (D) is correct because a sticking brake caliper on one side can cause the vehicle to pull to that side when braking. The caliper may not be releasing properly after the brake is applied, causing more braking force on that side and pulling the vehicle in that direction.
A4 – Question 2 of 5
A customer complains that their vehicle’s steering wheel shakes when driving at highway speeds. The vehicle does not pull to one side and the tires are in good condition. What is the most likely cause of this problem?
- The wheels are out of balance.
- The power steering pump is failing.
- The vehicle’s alignment is off.
- The steering gear is loose.
The correct answer is (A). The wheels are out of balance.
Explanation:
- Option (B) is incorrect because a failing power steering pump would typically cause difficulty steering or noise when turning the steering wheel, not a shake at highway speeds.
- Option (C) is incorrect because while an off alignment can cause a variety of issues, it would not typically cause the steering wheel to shake at highway speeds.
- Option (D) is incorrect because a loose steering gear could cause loose or sloppy steering, but it would not typically cause the steering wheel to shake at highway speeds.
- Option (A) is correct because wheels that are out of balance can cause the steering wheel to shake at certain speeds, typically around 50-70 mph. The imbalance causes the wheels to wobble as they rotate, which can be felt through the steering wheel.
A4 – Question 3 of 5
A customer complains that their vehicle’s steering feels loose and the vehicle wanders on the road. The tires are in good condition and the vehicle does not pull to one side. What is the most likely cause of this problem?
- The power steering fluid is low.
- The vehicle’s alignment is off.
- The steering linkage is worn.
- The brake rotors are warped.
The correct answer is (C). The steering linkage is worn.
Explanation:
- Option (A) is incorrect because low power steering fluid would typically cause difficulty steering or noise when turning the steering wheel, not loose steering or wandering.
- Option (B) is incorrect because while an off alignment can cause a variety of issues, it would not typically cause loose steering or wandering.
- Option (D) is incorrect because warped brake rotors would typically cause a pulsation in the brake pedal or a shake in the steering wheel when braking, not loose steering or wandering.
- Option (C) is correct because worn steering linkage can cause loose steering and wandering. The linkage connects the steering wheel to the wheels, and if it’s worn, it can allow the wheels to move independently of the steering wheel, resulting in a loose feel and wandering.
A4 – Question 4 of 5
A customer complains that their vehicle’s steering wheel is hard to turn. The vehicle does not make any noise when turning and the steering does not feel loose. What is the most likely cause of this problem?
- The power steering pump is failing.
- The wheels are out of balance.
- The steering gear is loose.
- The vehicle’s alignment is off.
The correct answer is (A). The power steering pump is failing.
Explanation:
- Option (B) is incorrect because wheels that are out of balance would typically cause the steering wheel to shake at certain speeds, not make it hard to turn.
- Option (C) is incorrect because a loose steering gear could cause loose or sloppy steering, but it would not typically make the steering wheel hard to turn.
- Option (D) is incorrect because while an off alignment can cause a variety of issues, it would not typically make the steering wheel hard to turn.
- Option (A) is correct because the power steering pump provides the hydraulic pressure needed to assist the steering. If it’s failing, it may not provide enough pressure, making the steering wheel hard to turn.
A4 – Question 5 of 5
Based on the diagram, which of the following is most likely to cause excessive body roll during cornering?
- A worn out control arm bushing.
- A faulty strut assembly.
- A worn out sway bar link.
- A loose tie rod end.
The correct answer is (C). A worn out sway bar link.
Explanation:
- Option (A) is incorrect because a worn out control arm bushing could cause a variety of suspension problems, including poor handling and uneven tire wear, but it would not typically cause excessive body roll during cornering.
- Option (B) is incorrect because a faulty strut assembly could cause a variety of suspension problems, including poor ride quality and uneven tire wear, but it would not typically cause excessive body roll during cornering.
- Option (D) is incorrect because a loose tie rod end could cause a variety of steering problems, including poor steering response and uneven tire wear, but it would not typically cause excessive body roll during cornering.
- Option (C) is correct because a worn out sway bar link can cause excessive body roll during cornering. The sway bar helps to reduce body roll during cornering. If the link that connects the sway bar to the suspension is worn out, it may not effectively control body roll.
A5 – Brakes Sample Questions
A5 – Question 1 of 5
A customer complains that their vehicle’s brakes squeal when applied. The brake pedal feels normal and the vehicle does not pull to one side when braking. What is the most likely cause of this problem?
- The brake pads are worn.
- The brake fluid is contaminated.
- The brake calipers are sticking.
- The brake rotors are warped.
The correct answer is (A). The brake pads are worn.
Explanation:
- Option (B) is incorrect because contaminated brake fluid would typically cause a spongy brake pedal feel or reduced braking performance, not squealing brakes.
- Option (C) is incorrect because sticking brake calipers could cause the vehicle to pull to one side when braking or cause the brakes to drag, not squeal.
- Option (D) is incorrect because warped brake rotors would typically cause a pulsation in the brake pedal or a shake in the steering wheel when braking, not squealing brakes.
- Option (A) is correct because worn brake pads can cause a squealing noise when applied. Many brake pads have a wear indicator that makes a squealing noise when the pads are worn to a certain point.
A5 – Question 2 of 5
A customer complains that their vehicle’s brake pedal feels spongy. The brakes do not make any noise and the vehicle does not pull to one side when braking. What is the most likely cause of this problem?
- The brake fluid is contaminated.
- The brake pads are worn.
- The brake calipers are sticking.
- The brake rotors are warped.
The correct answer is (A). The brake fluid is contaminated.
Explanation:
- Option (B) is incorrect because worn brake pads would typically cause a low brake pedal or a squealing noise, not a spongy pedal feel.
- Option (C) is incorrect because sticking brake calipers could cause the vehicle to pull to one side when braking or cause the brakes to drag, not a spongy pedal feel.
- Option (D) is incorrect because warped brake rotors would typically cause a pulsation in the brake pedal or a shake in the steering wheel when braking, not a spongy pedal feel.
- Option (A) is correct because contaminated brake fluid can cause a spongy brake pedal feel. If the fluid is contaminated with water or air, it can compress when the brakes are applied, resulting in a spongy feel.
A5 – Question 3 of 5
A customer complains that their vehicle’s brakes are dragging. The brake pedal feels normal and the brakes do not make any noise. What is the most likely cause of this problem?
- The brake rotors are warped.
- The brake calipers are sticking.
- The brake fluid is contaminated.
- The brake pads are worn.
The correct answer is (B). The brake calipers are sticking.
Explanation:
- Option (A) is incorrect because warped brake rotors would typically cause a pulsation in the brake pedal or a shake in the steering wheel when braking, not dragging brakes.
- Option (C) is incorrect because contaminated brake fluid would typically cause a spongy brake pedal feel or reduced braking performance, not dragging brakes.
- Option (D) is incorrect because worn brake pads would typically cause a low brake pedal or a squealing noise, not dragging brakes.
- Option (B) is correct because sticking brake calipers can cause the brakes to drag. The caliper may not be releasing properly after the brake is applied, causing the brakes to continue to apply pressure even when the pedal is released.
A5 – Question 4 of 5
A customer complains that their vehicle’s brake pedal pulsates when applied. The brakes do not make any noise and the vehicle does not pull to one side when braking. What is the most likely cause of this problem?
- The brake pads are worn.
- The brake fluid is contaminated.
- The brake calipers are sticking.
- The brake rotors are warped.
The correct answer is (D). The brake rotors are warped.
Explanation:
- Option (A) is incorrect because worn brake pads would typically cause a low brake pedal or a squealing noise, not a pulsating pedal.
- Option (B) is incorrect because contaminated brake fluid would typically cause a spongy brake pedal feel or reduced braking performance, not a pulsating pedal.
- Option (C) is incorrect because sticking brake calipers could cause the vehicle to pull to one side when braking or cause the brakes to drag, not a pulsating pedal.
- Option (D) is correct because warped brake rotors can cause the brake pedal to pulsate when applied. The uneven surface of the rotor causes the brake pads to move in and out as they pass over the high and low spots, resulting in a pulsation in the pedal.
A5 – Question 5 of 5
Based on the diagram, which of the following symptoms is most likely to be experienced by the driver when the brake pads are this worn?
- A high-pitched squealing noise when braking.
- A pulsating brake pedal.
- Brake fluid leaking onto the ground.
- A grinding noise when turning the steering wheel.
The correct answer is A. A high-pitched squealing noise when braking.
Explanation:
- Option A is correct because the brake pad wear indicator is designed to produce a high-pitched squealing noise when the brake pads are nearing the end of their service life. This noise serves as an audible warning to the driver that the brake pads need replacement.
- Option B is incorrect because a pulsating brake pedal is typically caused by a warped brake rotor, not worn brake pads.
- Option C is incorrect because brake fluid leaking onto the ground indicates a problem with the brake hydraulic system, not the brake pads.
- Option D is incorrect because a grinding noise when turning the steering wheel is more related to issues with the steering or suspension system, not the brake pads.
A6 – Electrical/Electronic Systems Sample Questions
A6 – Question 1 of 5
A customer complains that their vehicle’s battery keeps dying. The alternator has been tested and is working properly. The vehicle does not have any aftermarket accessories installed. What is the most likely cause of this problem?
- The battery is old and needs to be replaced.
- The vehicle has a short circuit.
- The vehicle’s ignition switch is faulty.
- The vehicle’s starter motor is drawing too much current.
The correct answer is (A). The battery is old and needs to be replaced.
Explanation:
- Option (B) is incorrect because a short circuit would typically cause a blown fuse or other electrical problems, not a dying battery.
- Option (C) is incorrect because a faulty ignition switch could cause a variety of issues, including a no-start condition, but it would not typically cause a dying battery.
- Option (D) is incorrect because a starter motor drawing too much current could cause a slow cranking condition or a no-start condition, but it would not typically cause a dying battery.
- Option (A) is correct because an old battery can lose its ability to hold a charge, resulting in a dying battery. If the alternator is working properly and there are no other electrical problems, the battery itself is the most likely cause.
A6 – Question 2 of 5
A customer complains that their vehicle’s headlights dim and brighten intermittently. The vehicle’s battery and alternator have been tested and are working properly. What is the most likely cause of this problem?
- The vehicle’s headlight switch is faulty.
- The vehicle’s battery cables are loose.
- The vehicle’s alternator belt is slipping.
- The vehicle’s ignition switch is faulty.
The correct answer is (B). The vehicle’s battery cables are loose.
Explanation:
- Option (A) is incorrect because a faulty headlight switch could cause the headlights to not work at all, but it would not typically cause them to dim and brighten intermittently.
- Option (C) is incorrect because a slipping alternator belt could cause the alternator to not charge properly, but if the alternator has been tested and is working properly, this is not the cause.
- Option (D) is incorrect because a faulty ignition switch could cause a variety of issues, including a no-start condition, but it would not typically cause the headlights to dim and brighten.
- Option (B) is correct because loose battery cables can cause intermittent electrical problems, including headlights that dim and brighten. The loose connection can cause the voltage to the headlights to fluctuate, resulting in the dimming and brightening.
A6 – Question 3 of 5
A customer complains that their vehicle’s interior lights do not work. The fuses have been checked and are not blown. The vehicle does not have any other electrical problems. What is the most likely cause of this problem?
- The vehicle’s battery is dead.
- The vehicle’s light switch is faulty.
- The vehicle’s alternator is not charging properly.
- The vehicle’s interior light bulbs are burnt out.
The correct answer is (D). The vehicle’s interior light bulbs are burnt out.
Explanation:
- Option (A) is incorrect because a dead battery would typically cause the vehicle to not start, not just the interior lights to not work.
- Option (C) is incorrect because an alternator not charging properly would typically cause the battery to die or other electrical problems, not just the interior lights to not work.
- Option (B) is incorrect because a faulty light switch could cause the interior lights to not turn on, but if all the interior lights are not working, it’s more likely that the bulbs are the problem.
- Option (D) is correct because if all the interior lights are not working and the fuses are not blown, it’s likely that the bulbs are burnt out. It’s possible for multiple bulbs to burn out at the same time, especially if they were all installed at the same time.
A6 – Question 4 of 5
A customer complains that their vehicle’s turn signals do not work. The fuses have been checked and are not blown. The vehicle does not have any other electrical problems. What is the most likely cause of this problem?
- The vehicle’s turn signal switch is faulty.
- The vehicle’s turn signal relay is faulty.
- The vehicle’s turn signal bulbs are burnt out.
- The vehicle’s battery is dead.
The correct answer is (B). The vehicle’s turn signal relay is faulty.
Explanation:
- Option (A) is incorrect because a faulty turn signal switch could cause the turn signals to not work, but it would typically affect only one side (left or right), not both.
- Option (C) is incorrect because while burnt out bulbs could cause the turn signals to not work, it’s unlikely that all the turn signal bulbs would burn out at the same time.
- Option (D) is incorrect because a dead battery would typically cause the vehicle to not start, not just the turn signals to not work.
- Option (B) is correct because a faulty turn signal relay can cause the turn signals to not work. The relay controls the on-off flashing of the turn signals, and if it’s faulty, it can prevent the turn signals from working at all.
A6 – Question 5 of 5
Based on the image, which of the following is the most probable cause for one headlight appearing dimmer?
- A faulty turn signal relay.
- A corroded ground connection for the dim headlight.
- A malfunctioning brake light switch.
- A blown fuse for the taillights.
The correct answer is B. A corroded ground connection for the dim headlight. Explanation:
- Option A is incorrect because the turn signal relay wouldn’t affect the brightness of the headlights.
- Option B is correct because a corroded ground connection can reduce the current flow to the headlight, making it appear dimmer.
- Option C is incorrect because the brake light switch wouldn’t affect the headlights.
- Option D is incorrect because a blown fuse for the taillights wouldn’t cause one headlight to appear dimmer.
A7 – Heating and Air Conditioning Sample Questions
A7 – Question 1 of 5
A customer complains that their vehicle’s air conditioning is not cooling properly. The A/C compressor has been tested and is working properly. The vehicle does not have any other issues. What is the most likely cause of this problem?
- The vehicle’s A/C system is low on refrigerant.
- The vehicle’s A/C compressor clutch is faulty.
- The vehicle’s A/C condenser is blocked.
- The vehicle’s A/C system has a leak.
The correct answer is (A). The vehicle’s A/C system is low on refrigerant.
Explanation:
- Option (B) is incorrect because a faulty A/C compressor clutch would typically prevent the compressor from operating, but the compressor has been tested and is working properly.
- Option (C) is incorrect because a blocked A/C condenser could cause the A/C system to not cool properly, but it would typically cause other issues as well, such as overheating.
- Option (D) is incorrect because while an A/C system leak could cause the system to not cool properly, it would typically result in a loss of refrigerant, which is covered by option (A).
- Option (A) is correct because if the A/C system is low on refrigerant, it will not cool properly. The refrigerant is what carries the heat away from the inside of the vehicle, so if there’s not enough of it, the system won’t be able to cool effectively.
A7 – Question 2 of 5
A customer complains that their vehicle’s heater is not working properly. The engine is reaching normal operating temperature and the vehicle does not have any other issues. What is the most likely cause of this problem?
- The vehicle’s thermostat is stuck open.
- The vehicle’s heater core is blocked.
- The vehicle’s coolant level is low.
- The vehicle’s blower motor is faulty.
The correct answer is (B). The vehicle’s heater core is blocked.
Explanation:
- Option (A) is incorrect because a thermostat stuck open would typically prevent the engine from reaching normal operating temperature, but the engine is reaching normal temperature in this case.
- Option (C) is incorrect because a low coolant level could cause the heater to not work properly, but it would typically cause other issues as well, such as overheating.
- Option (D) is incorrect because a faulty blower motor could prevent the heater from working, but it would typically cause the heater to not blow air at all, not just to not work properly.
- Option (B) is correct because a blocked heater core can prevent the heater from working properly. The heater core is like a small radiator that the coolant flows through. If it’s blocked, the hot coolant can’t flow through it to provide heat to the interior of the vehicle.
A7 – Question 3 of 5
A customer complains that their vehicle’s defroster is not clearing the windshield properly. The vehicle’s heater is working properly and the vehicle does not have any other issues. What is the most likely cause of this problem?
- The vehicle’s windshield is dirty on the inside.
- The vehicle’s A/C system is not working properly.
- The vehicle’s defroster vent is blocked.
- The vehicle’s blower motor is faulty.
The correct answer is (C). The vehicle’s defroster vent is blocked.
Explanation:
- Option (A) is incorrect because while a dirty windshield can make it harder for the defroster to clear the windshield, it would not typically prevent the defroster from working properly.
- Option (B) is incorrect because while a malfunctioning A/C system could affect the defroster’s ability to clear the windshield, the customer has not reported any issues with the A/C system.
- Option (D) is incorrect because a faulty blower motor could prevent the defroster from working, but it would typically cause the defroster to not blow air at all, not just to not work properly.
- Option (C) is correct because a blocked defroster vent can prevent the defroster from clearing the windshield properly. The defroster works by blowing air over the windshield to remove condensation. If the vent is blocked, the air can’t reach the windshield to do its job.
A7 – Question 4 of 5
A customer complains that their vehicle’s air conditioning is not cooling as well as it used to. The A/C compressor is engaging and the blower motor is working properly. What is the most likely cause of this problem?
- The vehicle’s A/C system is low on refrigerant.
- The vehicle’s A/C condenser fan is not working.
- The vehicle’s cabin air filter is clogged.
- The vehicle’s A/C compressor clutch is slipping.
The correct answer is (A). The vehicle’s A/C system is low on refrigerant.
Explanation:
- Option (B) is incorrect because a non-working A/C condenser fan would typically cause the A/C system to not cool at all, especially at idle or low speeds.
- Option (C) is incorrect because a clogged cabin air filter could reduce the airflow from the vents, but it would not typically cause the A/C to not cool as well.
- Option (D) is incorrect because a slipping A/C compressor clutch would typically prevent the compressor from engaging, but the compressor is engaging in this case.
- Option (A) is correct because if the A/C system is low on refrigerant, it will not cool as well as it used to. The refrigerant is what carries the heat away from the inside of the vehicle, so if there’s not enough of it, the system won’t be able to cool effectively.
A7 – Question 5 of 5
Based on the diagram, which component failure is most likely causing the ineffective defrosting?
- A malfunctioning A/C compressor.
- A clogged cabin air filter.
- A faulty blower motor.
- A leaking heater core.
The correct answer is A. A malfunctioning A/C compressor.
Explanation:
- Option A is correct because the A/C compressor plays a vital role in the defrosting process by removing moisture from the air. If it’s malfunctioning, the system won’t defrost effectively.
- Option B is incorrect because while a clogged cabin air filter can reduce airflow, it wouldn’t directly affect the defrosting process.
- Option C is incorrect because a faulty blower motor would prevent air from blowing, but the diagram suggests the system is turned on.
- Option D is incorrect because a leaking heater core would typically cause a sweet smell and fogging inside the vehicle, not ineffective defrosting.
A8 – Engine Performance Sample Questions
A8 – Question 1 of 5
A customer complains that their vehicle’s Check Engine Light (CEL) is on. The vehicle is running rough and has poor fuel economy. A scan tool reveals a code P0171 – System Too Lean (Bank 1). What is the most likely cause of this problem?
- The vehicle’s fuel pump is failing.
- The vehicle’s oxygen sensor (Bank 1, Sensor 1) is faulty.
- The vehicle’s mass air flow (MAF) sensor is dirty or faulty.
- The vehicle’s spark plugs are worn out.
The correct answer is (C). The vehicle’s mass air flow (MAF) sensor is dirty or faulty.
Explanation:
- Option (A) is incorrect because a failing fuel pump would typically cause a lack of power or a no-start condition, not a lean condition.
- Option (B) is incorrect because while a faulty oxygen sensor could cause a lean condition, it would typically set a specific oxygen sensor code, not a system lean code.
- Option (D) is incorrect because worn out spark plugs could cause a misfire or rough running, but they would not typically cause a lean condition.
- Option (C) is correct because a dirty or faulty MAF sensor can cause a lean condition. The MAF sensor measures the amount of air entering the engine. If it’s dirty or faulty, it can under-report the amount of air, causing the engine to run lean.
A8 – Question 2 of 5
A customer complains that their vehicle’s Check Engine Light (CEL) is on. The vehicle is running fine and does not have any noticeable issues. A scan tool reveals a code P0420 – Catalyst System Efficiency Below Threshold (Bank 1). What is the most likely cause of this problem?
- The vehicle’s catalytic converter (Bank 1) is failing.
- The vehicle’s oxygen sensor (Bank 1, Sensor 2) is faulty.
- The vehicle’s fuel injectors are clogged.
- The vehicle’s mass air flow (MAF) sensor is dirty or faulty.
The correct answer is (A). The vehicle’s catalytic converter (Bank 1) is failing.
Explanation:
- Option (B) is incorrect because while a faulty oxygen sensor could cause a P0420 code, it would typically set a specific oxygen sensor code, not a catalyst efficiency code.
- Option (C) is incorrect because clogged fuel injectors could cause a variety of issues, including misfires and poor fuel economy, but they would not typically cause a catalyst efficiency code.
- Option (D) is incorrect because a dirty or faulty MAF sensor could cause a variety of issues, including lean or rich conditions, but it would not typically cause a catalyst efficiency code.
- Option (A) is correct because a failing catalytic converter can cause a P0420 code. The catalytic converter is responsible for reducing harmful emissions. If it’s not working efficiently, it can set a P0420 code.
A8 – Question 3 of 5
A customer complains that their vehicle’s Check Engine Light (CEL) is on. The vehicle is hard to start and runs rough when it’s cold, but runs fine when it’s warm. A scan tool reveals a code P0113 – Intake Air Temperature (IAT) Sensor 1 Circuit High Input. What is the most likely cause of this problem?
- The vehicle’s intake air temperature (IAT) sensor is faulty.
- The vehicle’s coolant temperature sensor is faulty.
- The vehicle’s fuel pump is failing.
- The vehicle’s spark plugs are worn out.
The correct answer is (A). The vehicle’s intake air temperature (IAT) sensor is faulty.
Explanation:
- Option (B) is incorrect because while a faulty coolant temperature sensor could cause hard starting and rough running when cold, it would typically set a specific coolant temperature sensor code, not an intake air temperature sensor code.
- Option (C) is incorrect because a failing fuel pump would typically cause a lack of power or a no-start condition, not a specific intake air temperature sensor code.
- Option (D) is incorrect because worn out spark plugs could cause a misfire or rough running, but they would not typically cause a specific intake air temperature sensor code.
- Option (A) is correct because a faulty IAT sensor can cause a P0113 code. The IAT sensor measures the temperature of the air entering the engine. If it’s faulty, it can send incorrect information to the engine control module (ECM), causing the engine to run poorly when it’s cold.
A8 – Question 4 of 5
A customer complains that their vehicle’s Check Engine Light (CEL) is on. The vehicle has poor fuel economy and lacks power. A scan tool reveals a code P0172 – System Too Rich (Bank 1). What is the most likely cause of this problem?
- The vehicle’s fuel pressure regulator is faulty.
- The vehicle’s oxygen sensor (Bank 1, Sensor 1) is faulty.
- The vehicle’s mass air flow (MAF) sensor is dirty or faulty.
- The vehicle’s spark plugs are worn out.
The correct answer is (C). The vehicle’s mass air flow (MAF) sensor is dirty or faulty.
Explanation:
- Option (A) is incorrect because a faulty fuel pressure regulator could cause a rich condition, but it would typically also cause other issues, such as hard starting or black smoke from the exhaust.
- Option (B) is incorrect because while a faulty oxygen sensor could cause a rich condition, it would typically set a specific oxygen sensor code, not a system rich code.
- Option (D) is incorrect because worn out spark plugs could cause a misfire or lack of power, but they would not typically cause a rich condition.
- Option (C) is correct because a dirty or faulty MAF sensor can cause a rich condition. The MAF sensor measures the amount of air entering the engine. If it’s dirty or faulty, it can over-report the amount of air, causing the engine to run rich.
A8 – Question 5 of 5
Which of the following issues, as depicted in the diagram, is most likely causing the erratic waveform from the throttle position sensor?
- A faulty accelerator pedal position sensor.
- A malfunctioning electronic throttle control module.
- A worn-out throttle body.
- A failing throttle position sensor.
The correct answer is: D. A failing throttle position sensor.
Explanation:
The throttle position sensor (TPS) monitors the position of the throttle plate inside the throttle body and sends this information to the engine control module. An erratic waveform from the TPS indicates issues with its functionality.
- Option A is incorrect because while the accelerator pedal position sensor communicates the driver’s intent to the engine control module, an issue with this sensor would typically result in a lack of throttle response or inconsistent acceleration, not an erratic TPS waveform.
- Option B is incorrect because a malfunctioning electronic throttle control module would affect the overall operation of the throttle system, potentially leading to reduced engine power or a throttle lag, but it wouldn’t directly cause an erratic waveform from the TPS.
- Option C is incorrect because a worn-out throttle body might cause issues like a rough idle or stalling, but it wouldn’t directly result in an erratic TPS waveform.
- Option D is correct because a failing TPS would provide inconsistent readings to the engine control module, as evidenced by the erratic waveform in the diagram.
Well done!
You have completed the Sample Questions section.
The complete iPREP course includes full test simulations with detailed explanations and study guides.
‘…TESTS THAT ACTUALLY HELP’
In the first 30 minutes of use I have learned so much more than skipping along the internet looking for free content. Don’t waste you time, pay and get tests that actually help.
Richard Rodgers
January 28, 2020 at 7:49 PM